Finding extremals which arise from a non-linear third order ODE

The name of the pictureThe name of the pictureThe name of the pictureClash Royale CLAN TAG#URR8PPP











up vote
2
down vote

favorite
1












I'm trying to determine the extremals of the functional $F(x,y,y',y'') = (y')^2 + (y'')^2$.



I know that in this case the first integral satisfies



$$F - y' left( F_y' - fracmathrmdmathrmdxF_y''right)-y''F_y''=k$$



where $k$ is a constant.



For the aforementioned functional, I can easily see that $F_y' = 2y'$ and that $F_y'' = 2y''$, and furthermore $fracmathrmdmathrmdxF_y''=2y'''$.



If I put this into the equation satisfied by the extremal, we get
$$(y')^2 + (y'')^2 -y'(2y' - 2y''') - y''(2y'')=k$$



and it is then possible to re-write this as



$$(y')^2 + (y'')^2 = 2y'y''' - k$$



I am unsure what to do next. Is there a trick that I can use to solve this nonlinear third order ODE? I tried integrating both sides between 0 and 1 (as those are the two $x$-values that the boundary conditions specify) as I can see that the functional appears on the left hand side of the equation, but the exercise I am doing is asking for the extremals which implies that I need to find $y(x)$ rather than what the stationary value of the integral is.







share|cite|improve this question























    up vote
    2
    down vote

    favorite
    1












    I'm trying to determine the extremals of the functional $F(x,y,y',y'') = (y')^2 + (y'')^2$.



    I know that in this case the first integral satisfies



    $$F - y' left( F_y' - fracmathrmdmathrmdxF_y''right)-y''F_y''=k$$



    where $k$ is a constant.



    For the aforementioned functional, I can easily see that $F_y' = 2y'$ and that $F_y'' = 2y''$, and furthermore $fracmathrmdmathrmdxF_y''=2y'''$.



    If I put this into the equation satisfied by the extremal, we get
    $$(y')^2 + (y'')^2 -y'(2y' - 2y''') - y''(2y'')=k$$



    and it is then possible to re-write this as



    $$(y')^2 + (y'')^2 = 2y'y''' - k$$



    I am unsure what to do next. Is there a trick that I can use to solve this nonlinear third order ODE? I tried integrating both sides between 0 and 1 (as those are the two $x$-values that the boundary conditions specify) as I can see that the functional appears on the left hand side of the equation, but the exercise I am doing is asking for the extremals which implies that I need to find $y(x)$ rather than what the stationary value of the integral is.







    share|cite|improve this question





















      up vote
      2
      down vote

      favorite
      1









      up vote
      2
      down vote

      favorite
      1






      1





      I'm trying to determine the extremals of the functional $F(x,y,y',y'') = (y')^2 + (y'')^2$.



      I know that in this case the first integral satisfies



      $$F - y' left( F_y' - fracmathrmdmathrmdxF_y''right)-y''F_y''=k$$



      where $k$ is a constant.



      For the aforementioned functional, I can easily see that $F_y' = 2y'$ and that $F_y'' = 2y''$, and furthermore $fracmathrmdmathrmdxF_y''=2y'''$.



      If I put this into the equation satisfied by the extremal, we get
      $$(y')^2 + (y'')^2 -y'(2y' - 2y''') - y''(2y'')=k$$



      and it is then possible to re-write this as



      $$(y')^2 + (y'')^2 = 2y'y''' - k$$



      I am unsure what to do next. Is there a trick that I can use to solve this nonlinear third order ODE? I tried integrating both sides between 0 and 1 (as those are the two $x$-values that the boundary conditions specify) as I can see that the functional appears on the left hand side of the equation, but the exercise I am doing is asking for the extremals which implies that I need to find $y(x)$ rather than what the stationary value of the integral is.







      share|cite|improve this question











      I'm trying to determine the extremals of the functional $F(x,y,y',y'') = (y')^2 + (y'')^2$.



      I know that in this case the first integral satisfies



      $$F - y' left( F_y' - fracmathrmdmathrmdxF_y''right)-y''F_y''=k$$



      where $k$ is a constant.



      For the aforementioned functional, I can easily see that $F_y' = 2y'$ and that $F_y'' = 2y''$, and furthermore $fracmathrmdmathrmdxF_y''=2y'''$.



      If I put this into the equation satisfied by the extremal, we get
      $$(y')^2 + (y'')^2 -y'(2y' - 2y''') - y''(2y'')=k$$



      and it is then possible to re-write this as



      $$(y')^2 + (y'')^2 = 2y'y''' - k$$



      I am unsure what to do next. Is there a trick that I can use to solve this nonlinear third order ODE? I tried integrating both sides between 0 and 1 (as those are the two $x$-values that the boundary conditions specify) as I can see that the functional appears on the left hand side of the equation, but the exercise I am doing is asking for the extremals which implies that I need to find $y(x)$ rather than what the stationary value of the integral is.









      share|cite|improve this question










      share|cite|improve this question




      share|cite|improve this question









      asked Jul 18 at 11:28









      omegaSQU4RED

      1936




      1936




















          1 Answer
          1






          active

          oldest

          votes

















          up vote
          2
          down vote



          accepted










          Calling



          $$
          F(x,y,y',y'')= (y')^2+(y'')^2
          $$



          we have according to Euler-Lagrange conditions



          $$
          F_y-left(F_y'right)'+left(F_y''right)'' = -2(y^(4)-y'')=0
          $$



          hence



          $$
          y = C_1e^x+C_2e^-x+C_3 x+ C_4
          $$






          share|cite|improve this answer























          • Thanks a lot for this! I ended up using the formula for the first integral to try and find the extremal rather than simply solving the Euler-Lagrange equation... now it seems so obvious. Cheers!
            – omegaSQU4RED
            Jul 18 at 16:49










          • Actually, you might want to double-check the Euler-Lagrange condition and plugging all the derivatives into it. I ended up getting $F_y = 0$, $(F_y')' = 2y''$ and $(F_y'')'' = 2y'''$ which, when put into the condition, ends up with $$2(y'''' - y'') = 0$$ which has general solution $$y = C_1 e^x + C_2 e^-x + C_3 x + C_4$$. This seems to make sense from the complete version of the problem in the book - the boundary conditions do not involve "nice" angular numbers in $x$. Still thanks though - your approache was nonetheless a lot more elegant than mine :-)
            – omegaSQU4RED
            Jul 18 at 17:06










          • @omegaSQU4RED Thanks for the correction. I had made a change of sign.
            – Cesareo
            Jul 18 at 17:28










          Your Answer




          StackExchange.ifUsing("editor", function ()
          return StackExchange.using("mathjaxEditing", function ()
          StackExchange.MarkdownEditor.creationCallbacks.add(function (editor, postfix)
          StackExchange.mathjaxEditing.prepareWmdForMathJax(editor, postfix, [["$", "$"], ["\\(","\\)"]]);
          );
          );
          , "mathjax-editing");

          StackExchange.ready(function()
          var channelOptions =
          tags: "".split(" "),
          id: "69"
          ;
          initTagRenderer("".split(" "), "".split(" "), channelOptions);

          StackExchange.using("externalEditor", function()
          // Have to fire editor after snippets, if snippets enabled
          if (StackExchange.settings.snippets.snippetsEnabled)
          StackExchange.using("snippets", function()
          createEditor();
          );

          else
          createEditor();

          );

          function createEditor()
          StackExchange.prepareEditor(
          heartbeatType: 'answer',
          convertImagesToLinks: true,
          noModals: false,
          showLowRepImageUploadWarning: true,
          reputationToPostImages: 10,
          bindNavPrevention: true,
          postfix: "",
          noCode: true, onDemand: true,
          discardSelector: ".discard-answer"
          ,immediatelyShowMarkdownHelp:true
          );



          );








           

          draft saved


          draft discarded


















          StackExchange.ready(
          function ()
          StackExchange.openid.initPostLogin('.new-post-login', 'https%3a%2f%2fmath.stackexchange.com%2fquestions%2f2855492%2ffinding-extremals-which-arise-from-a-non-linear-third-order-ode%23new-answer', 'question_page');

          );

          Post as a guest






























          1 Answer
          1






          active

          oldest

          votes








          1 Answer
          1






          active

          oldest

          votes









          active

          oldest

          votes






          active

          oldest

          votes








          up vote
          2
          down vote



          accepted










          Calling



          $$
          F(x,y,y',y'')= (y')^2+(y'')^2
          $$



          we have according to Euler-Lagrange conditions



          $$
          F_y-left(F_y'right)'+left(F_y''right)'' = -2(y^(4)-y'')=0
          $$



          hence



          $$
          y = C_1e^x+C_2e^-x+C_3 x+ C_4
          $$






          share|cite|improve this answer























          • Thanks a lot for this! I ended up using the formula for the first integral to try and find the extremal rather than simply solving the Euler-Lagrange equation... now it seems so obvious. Cheers!
            – omegaSQU4RED
            Jul 18 at 16:49










          • Actually, you might want to double-check the Euler-Lagrange condition and plugging all the derivatives into it. I ended up getting $F_y = 0$, $(F_y')' = 2y''$ and $(F_y'')'' = 2y'''$ which, when put into the condition, ends up with $$2(y'''' - y'') = 0$$ which has general solution $$y = C_1 e^x + C_2 e^-x + C_3 x + C_4$$. This seems to make sense from the complete version of the problem in the book - the boundary conditions do not involve "nice" angular numbers in $x$. Still thanks though - your approache was nonetheless a lot more elegant than mine :-)
            – omegaSQU4RED
            Jul 18 at 17:06










          • @omegaSQU4RED Thanks for the correction. I had made a change of sign.
            – Cesareo
            Jul 18 at 17:28














          up vote
          2
          down vote



          accepted










          Calling



          $$
          F(x,y,y',y'')= (y')^2+(y'')^2
          $$



          we have according to Euler-Lagrange conditions



          $$
          F_y-left(F_y'right)'+left(F_y''right)'' = -2(y^(4)-y'')=0
          $$



          hence



          $$
          y = C_1e^x+C_2e^-x+C_3 x+ C_4
          $$






          share|cite|improve this answer























          • Thanks a lot for this! I ended up using the formula for the first integral to try and find the extremal rather than simply solving the Euler-Lagrange equation... now it seems so obvious. Cheers!
            – omegaSQU4RED
            Jul 18 at 16:49










          • Actually, you might want to double-check the Euler-Lagrange condition and plugging all the derivatives into it. I ended up getting $F_y = 0$, $(F_y')' = 2y''$ and $(F_y'')'' = 2y'''$ which, when put into the condition, ends up with $$2(y'''' - y'') = 0$$ which has general solution $$y = C_1 e^x + C_2 e^-x + C_3 x + C_4$$. This seems to make sense from the complete version of the problem in the book - the boundary conditions do not involve "nice" angular numbers in $x$. Still thanks though - your approache was nonetheless a lot more elegant than mine :-)
            – omegaSQU4RED
            Jul 18 at 17:06










          • @omegaSQU4RED Thanks for the correction. I had made a change of sign.
            – Cesareo
            Jul 18 at 17:28












          up vote
          2
          down vote



          accepted







          up vote
          2
          down vote



          accepted






          Calling



          $$
          F(x,y,y',y'')= (y')^2+(y'')^2
          $$



          we have according to Euler-Lagrange conditions



          $$
          F_y-left(F_y'right)'+left(F_y''right)'' = -2(y^(4)-y'')=0
          $$



          hence



          $$
          y = C_1e^x+C_2e^-x+C_3 x+ C_4
          $$






          share|cite|improve this answer















          Calling



          $$
          F(x,y,y',y'')= (y')^2+(y'')^2
          $$



          we have according to Euler-Lagrange conditions



          $$
          F_y-left(F_y'right)'+left(F_y''right)'' = -2(y^(4)-y'')=0
          $$



          hence



          $$
          y = C_1e^x+C_2e^-x+C_3 x+ C_4
          $$







          share|cite|improve this answer















          share|cite|improve this answer



          share|cite|improve this answer








          edited Jul 18 at 17:25


























          answered Jul 18 at 13:17









          Cesareo

          5,7722412




          5,7722412











          • Thanks a lot for this! I ended up using the formula for the first integral to try and find the extremal rather than simply solving the Euler-Lagrange equation... now it seems so obvious. Cheers!
            – omegaSQU4RED
            Jul 18 at 16:49










          • Actually, you might want to double-check the Euler-Lagrange condition and plugging all the derivatives into it. I ended up getting $F_y = 0$, $(F_y')' = 2y''$ and $(F_y'')'' = 2y'''$ which, when put into the condition, ends up with $$2(y'''' - y'') = 0$$ which has general solution $$y = C_1 e^x + C_2 e^-x + C_3 x + C_4$$. This seems to make sense from the complete version of the problem in the book - the boundary conditions do not involve "nice" angular numbers in $x$. Still thanks though - your approache was nonetheless a lot more elegant than mine :-)
            – omegaSQU4RED
            Jul 18 at 17:06










          • @omegaSQU4RED Thanks for the correction. I had made a change of sign.
            – Cesareo
            Jul 18 at 17:28
















          • Thanks a lot for this! I ended up using the formula for the first integral to try and find the extremal rather than simply solving the Euler-Lagrange equation... now it seems so obvious. Cheers!
            – omegaSQU4RED
            Jul 18 at 16:49










          • Actually, you might want to double-check the Euler-Lagrange condition and plugging all the derivatives into it. I ended up getting $F_y = 0$, $(F_y')' = 2y''$ and $(F_y'')'' = 2y'''$ which, when put into the condition, ends up with $$2(y'''' - y'') = 0$$ which has general solution $$y = C_1 e^x + C_2 e^-x + C_3 x + C_4$$. This seems to make sense from the complete version of the problem in the book - the boundary conditions do not involve "nice" angular numbers in $x$. Still thanks though - your approache was nonetheless a lot more elegant than mine :-)
            – omegaSQU4RED
            Jul 18 at 17:06










          • @omegaSQU4RED Thanks for the correction. I had made a change of sign.
            – Cesareo
            Jul 18 at 17:28















          Thanks a lot for this! I ended up using the formula for the first integral to try and find the extremal rather than simply solving the Euler-Lagrange equation... now it seems so obvious. Cheers!
          – omegaSQU4RED
          Jul 18 at 16:49




          Thanks a lot for this! I ended up using the formula for the first integral to try and find the extremal rather than simply solving the Euler-Lagrange equation... now it seems so obvious. Cheers!
          – omegaSQU4RED
          Jul 18 at 16:49












          Actually, you might want to double-check the Euler-Lagrange condition and plugging all the derivatives into it. I ended up getting $F_y = 0$, $(F_y')' = 2y''$ and $(F_y'')'' = 2y'''$ which, when put into the condition, ends up with $$2(y'''' - y'') = 0$$ which has general solution $$y = C_1 e^x + C_2 e^-x + C_3 x + C_4$$. This seems to make sense from the complete version of the problem in the book - the boundary conditions do not involve "nice" angular numbers in $x$. Still thanks though - your approache was nonetheless a lot more elegant than mine :-)
          – omegaSQU4RED
          Jul 18 at 17:06




          Actually, you might want to double-check the Euler-Lagrange condition and plugging all the derivatives into it. I ended up getting $F_y = 0$, $(F_y')' = 2y''$ and $(F_y'')'' = 2y'''$ which, when put into the condition, ends up with $$2(y'''' - y'') = 0$$ which has general solution $$y = C_1 e^x + C_2 e^-x + C_3 x + C_4$$. This seems to make sense from the complete version of the problem in the book - the boundary conditions do not involve "nice" angular numbers in $x$. Still thanks though - your approache was nonetheless a lot more elegant than mine :-)
          – omegaSQU4RED
          Jul 18 at 17:06












          @omegaSQU4RED Thanks for the correction. I had made a change of sign.
          – Cesareo
          Jul 18 at 17:28




          @omegaSQU4RED Thanks for the correction. I had made a change of sign.
          – Cesareo
          Jul 18 at 17:28












           

          draft saved


          draft discarded


























           


          draft saved


          draft discarded














          StackExchange.ready(
          function ()
          StackExchange.openid.initPostLogin('.new-post-login', 'https%3a%2f%2fmath.stackexchange.com%2fquestions%2f2855492%2ffinding-extremals-which-arise-from-a-non-linear-third-order-ode%23new-answer', 'question_page');

          );

          Post as a guest













































































          Comments

          Popular posts from this blog

          Color the edges and diagonals of a regular polygon

          Relationship between determinant of matrix and determinant of adjoint?

          What is the equation of a 3D cone with generalised tilt?